Difference between revisions of "2021 Fall AMC 10A Problems/Problem 12"

m (Problem)
(Corrected the sign error, and reformatted. Made Sol 1 more concise.)
Line 1: Line 1:
 
==Problem==
 
==Problem==
The base-nine representation of the number <math>N</math> is <math>27,006,000,052_{\text{nine}}.</math> What is the remainder when <math>N</math> is divided by <math>5?</math>
+
The base-nine representation of the number <math>N</math> is <math>27{,}006{,}000{,}052_{\text{nine}}.</math> What is the remainder when <math>N</math> is divided by <math>5?</math>
  
 
<math>\textbf{(A) } 0\qquad\textbf{(B) } 1\qquad\textbf{(C) } 2\qquad\textbf{(D) } 3\qquad\textbf{(E) }4</math>
 
<math>\textbf{(A) } 0\qquad\textbf{(B) } 1\qquad\textbf{(C) } 2\qquad\textbf{(D) } 3\qquad\textbf{(E) }4</math>
  
==Solution==
+
==Solution 1==
Using module rules, we can find the remainder:
+
Recall that <math>9\equiv-1\pmod{5}.</math> We have
 
+
<cmath>\begin{align*}
<math>27,006,000,052_9 = 2(9^{10})+7(9^9)+6(9^6)+5(9^1)+2</math>
+
27{,}006{,}000{,}052_9 &= 2\cdot9^{10} + 7\cdot9^9 + 6\cdot9^6 + 5\cdot9 + 2 \\
 
+
&\equiv 2\cdot(-1)^{10} + 7\cdot(-1)^9 + 6\cdot(-1)^6 + 5\cdot(-1) + 2 &&\pmod{5} \\
<math>2(9^{10})+7(9^9)+6(9^6)+5(9^1)+2\equiv 2({-}1^{10})+7({-}1^9)+6({-}1^6)+5({-}1^1)+2 (\text{mod }5)</math>
+
&= 2-7+6-5+2 \\
 
+
&= -2 \\
<math>2({-}1^{10})+7({-}1^9)+6({-}1^6)+5({-}1^1)+2\equiv 2-7+6-5+2(\text{mod }5)</math>
+
&\equiv \boxed{\textbf{(D) } 3} &&\pmod{5}.
 
+
\end{align*}</cmath>
<math>2-7+6-5+2\equiv -2(\text{mod }5)</math>
+
-Aidensharp ~MRENTHUSIASM
 
 
<math>-2\equiv 3(\text{mod }5)</math>
 
 
 
Thus, the answer is <math>\boxed{\textbf{(D)}\ 3}</math>.
 
 
 
-Aidensharp
 
  
 
==Solution 2==
 
==Solution 2==
Line 28: Line 22:
 
Simplifying, <math>-2 \mod 5 \implies 3 \mod 5.</math> So, the answer is <math>\boxed{3}.</math>
 
Simplifying, <math>-2 \mod 5 \implies 3 \mod 5.</math> So, the answer is <math>\boxed{3}.</math>
  
- kante314
+
-kante314
 +
 
 +
==See Also==
 +
{{AMC10 box|year=2021 Fall|ab=A|num-b=11|num-a=13}}
 +
{{MAA Notice}}

Revision as of 21:26, 22 November 2021

Problem

The base-nine representation of the number $N$ is $27{,}006{,}000{,}052_{\text{nine}}.$ What is the remainder when $N$ is divided by $5?$

$\textbf{(A) } 0\qquad\textbf{(B) } 1\qquad\textbf{(C) } 2\qquad\textbf{(D) } 3\qquad\textbf{(E) }4$

Solution 1

Recall that $9\equiv-1\pmod{5}.$ We have \begin{align*} 27{,}006{,}000{,}052_9 &= 2\cdot9^{10} + 7\cdot9^9 + 6\cdot9^6 + 5\cdot9 + 2 \\ &\equiv 2\cdot(-1)^{10} + 7\cdot(-1)^9 + 6\cdot(-1)^6 + 5\cdot(-1) + 2 &&\pmod{5} \\ &= 2-7+6-5+2 \\ &= -2 \\ &\equiv \boxed{\textbf{(D) } 3} &&\pmod{5}. \end{align*} -Aidensharp ~MRENTHUSIASM

Solution 2

We convert this into base $10,$ so \[2 \cdot 9^{10}+7 \cdot 9^9+6 \cdot 9^6+5 \cdot 9+2\] Notice that $9 \equiv -1 \mod 5,$ \[2 \cdot (-1)^10+7 \cdot (-1)^9+6 \cdot (-1)^6+5 \cdot (-1)+2=2-7+6-5+2\] Simplifying, $-2 \mod 5 \implies 3 \mod 5.$ So, the answer is $\boxed{3}.$

-kante314

See Also

2021 Fall AMC 10A (ProblemsAnswer KeyResources)
Preceded by
Problem 11
Followed by
Problem 13
1 2 3 4 5 6 7 8 9 10 11 12 13 14 15 16 17 18 19 20 21 22 23 24 25
All AMC 10 Problems and Solutions

The problems on this page are copyrighted by the Mathematical Association of America's American Mathematics Competitions. AMC logo.png